1answer.
Ask question
Login Signup
Ask question
All categories
  • English
  • Mathematics
  • Social Studies
  • Business
  • History
  • Health
  • Geography
  • Biology
  • Physics
  • Chemistry
  • Computers and Technology
  • Arts
  • World Languages
  • Spanish
  • French
  • German
  • Advanced Placement (AP)
  • SAT
  • Medicine
  • Law
  • Engineering
Kobotan [32]
3 years ago
12

5,049 +48=105

Mathematics
1 answer:
Alborosie3 years ago
4 0
<h2>Solution (a) :-</h2>

Let the cost of the less expensive comic book be x .

Then :-

1284 - 11x = x

Using this equation , we can find the cost of the less expensive comic book .

Price of the less expensive book :-

1284 - 11x = x

11x + x = 1284

12x = 1284

x =  \frac{1284}{12}

x = 107

Cost of the less expensive book is 107 .

Therefore , the epice of the less expensive book = $ 107 .

<h2>Solution (b) :-</h2>

Cost of the more expensive book = 11x

Which means :-

1284 - x = 11x

11x + x = 1284

12x = 1284

x = 107

11x = 11 \times 107

= 1177

Therefore , the price of the more expensive book = $ 1177 .

You might be interested in
Anna wants to take fitness classes. She compares two gyms to determine which would be the best deal for her. Fit Fast charges a
mestny [16]
Fit Fast: a set feet per class => y = Ax

Stepping Up: a monthly fee plus an additioal fee per class => h = Bx + C

You can discard the second and the fourth systems because  they do not have the form established from the statement.

The first system produce an obvious result given that is represents an option that is always better than the other 5.5x will be lower than 7.5x + 10 for any positive value of x, and so there is no need to make any comparission.

The third system is

y = 7.5x and y = 5.5x + 10 which need to be solved to determine when one rate is more convenient than the other.

Answer: y = 7.5x and y = 5..5x + 10



4 0
3 years ago
Read 2 more answers
Answer numbers 16-18
lana66690 [7]
Number 16- 216
number 17- 180
number 18- 252
4 0
3 years ago
50 points! Mhanifa please help! Everyone is putting random answers when I post a question and I need this done asap! DONT ANSWER
Genrish500 [490]

Answer:

  • x = 26°

Step-by-step explanation:

The first two questions are answered here

  • brainly.com/question/21794653
<h3>Question 3</h3>

Use the equation for exterior angle

<u>Exterior angle  = sum of non-adjacent interior angles</u>

  • 5x - 2 = 2x + 76
  • 5x - 2x = 76 + 2
  • 3x = 78
  • x = 26
6 0
3 years ago
Read 2 more answers
Solve the following equation for A. Be sure to take into account whether a letter is
Sati [7]

Answer:

4/a

Step-by-step explanation:

8 0
2 years ago
Find the equation of the line passing through the point (4,−1) that is parallel to the line 2x−3y=9 Find the slope of the line 2
AnnZ [28]

Answer:

Step-by-step explanation:

2x - 3y = 9

-3y = -2x + 9

y=\frac{-2}{-3}x + \frac{9}{-3}\\\\y=\frac{2}{3}x-3\\

Parallel lines have same slope.So,

Slope m = 2/3

(4 , -1)

Equation: y - y1 = m(x - x1)

y-[-1]=\frac{2}{3}(x - 4)\\\\y+1=\frac{2}{3}*x - \frac{2}{3}*4\\\\y+1=\frac{2}{3}x-\frac{8}{3}\\\\y=\frac{2}{3}x-\frac{8}{3}-1\\\\y=\frac{2}{3}x-\frac{8}{3}-\frac{3}{3}\\\\y=\frac{2}{3}x-\frac{11}{3}

b = -11/3

5 0
3 years ago
Read 2 more answers
Other questions:
  • Round to the nearest tenth. 8.54
    12·2 answers
  • Calculate the expected value E(X) of the given random variable X. X is the higher number when two dice are rolled, or the common
    6·1 answer
  • How many ounces of trail mix are in a bag that weighs 0.681 kilograms? Input only numeric values. (1 pound = 0.454 kg and 1 poun
    15·1 answer
  • For the graph below, what should the domain be so that the function is at least 300?
    14·2 answers
  • Billy Idell is trying to determine how many units of two types of lawn mowers to produce each day. One of these is the Standard
    15·1 answer
  • Red roses come 3 to a package, and white roses come 5 to a package. If an equal number of red and white roses are wanted to make
    5·1 answer
  • Can all one-variable inequalities be simplified to a two-step inequality?
    14·1 answer
  • He is pulling 400 feet over level ground by exerting a constant 25-pound force on the bar that makes an angle of 30° with the gr
    12·2 answers
  • Solve the equation. x^2=1/49
    10·1 answer
  • Car A is traveling east at a steady speed of 60 miles per hour. After 2 hours, it is 150 miles east of Johnstown.
    13·2 answers
Add answer
Login
Not registered? Fast signup
Signup
Login Signup
Ask question!